Proving a Formula: Maximum Root Count for Polynomials

  • Thread starter Thread starter chocolatelover
  • Start date Start date
  • Tags Tags
    Formula
Click For Summary
A polynomial p(x) of degree n can have at most n roots, as established by the fundamental theorem of algebra. If a polynomial has n+1 distinct roots, it can be factored into linear components, leading to a contradiction when reaching a linear polynomial with two distinct roots. The discussion emphasizes proving this by considering the case when n=1, illustrating that a linear polynomial cannot possess two roots. Participants seek clarification on the proof method, with suggestions to approach the problem through contrapositive reasoning. The conversation highlights the importance of understanding polynomial factorization in relation to root count.
chocolatelover
Messages
238
Reaction score
0

Homework Statement


Prove that if p(x)=anx^n +an-1x^n-1+...a0, where a0,..., "an" ε reals, is a polynomial, then p can have at most n roots.


Homework Equations





The Attempt at a Solution



C ε R is a root of a polynomial p if p(c)=0. If c is a root of p, then x-c is a factor of p.

I'm not sure where to go from here. I think it would probably be the easiest to prove this by proving the contrapositive as being false. Could someone please give me a hint or show me where to go from here?

Thank you very much
 
Physics news on Phys.org
Sure. Suppose the polynomial has n+1 different roots. c1,c2,...cn+1. Since c1 is a root the polynomial p(x) can be factored (x-c1)*p1(x) where p1 has degree n-1. The other c's must be roots of p1(x) since they aren't roots of (x-c1). Continue in this way until you reach degree 1. Now you have a linear polynomial with two different roots. Possible?
 
Thank you very much

Would it be somthing like this?

(p1x)^(n-1)(x-c2)(x-c3)^(n) :confused:

Thank you
 
Last edited:
No, that's not clear at all. Start by proving if n=1 then the polynomial can't have 2 roots. Ok?
 
Thank you very much

Regards
 
Question: A clock's minute hand has length 4 and its hour hand has length 3. What is the distance between the tips at the moment when it is increasing most rapidly?(Putnam Exam Question) Answer: Making assumption that both the hands moves at constant angular velocities, the answer is ## \sqrt{7} .## But don't you think this assumption is somewhat doubtful and wrong?

Similar threads

  • · Replies 1 ·
Replies
1
Views
2K
  • · Replies 1 ·
Replies
1
Views
1K
  • · Replies 22 ·
Replies
22
Views
866
Replies
7
Views
1K
  • · Replies 2 ·
Replies
2
Views
2K
  • · Replies 3 ·
Replies
3
Views
1K
Replies
1
Views
1K
  • · Replies 4 ·
Replies
4
Views
4K
  • · Replies 1 ·
Replies
1
Views
2K
  • · Replies 3 ·
Replies
3
Views
2K